Download as pdf or txt
Download as pdf or txt
You are on page 1of 12

03.

Motion in Plane
(a) Scalar and Vector Quantity 3. A and B are vectors such that A + B =

1. If A, B are perpendicular vectors A - B . Then, the angle between them is

A = 5iˆ + 7jˆ - 3kˆ (a) 90º (b) 60º


(c) 45º (d) 0º
B = 2iˆ + 2jˆ - ckˆ . EAMCET-1993
The value of c is Ans. (a) : Given,
(a) –2 (b) 8 | A + B |=| A − B |
(c) –7 (d) –8
Squaring both sides,
EAMCET-1991
| A + B |2 =| A − B |2
Ans. (d) : A = 5iˆ + 7ˆj − 3kˆ
A 2 + B2 + 2A ⋅ B = A 2 + B2 − 2A ⋅ B
B = 2iˆ + 2ˆj − ckˆ
For perpendicular vectors 4A ⋅ B = 0
A⋅B = 0 A⋅B = 0

( )( )
5iˆ + 7ˆj − 3kˆ ⋅ 2iˆ + 2ˆj − ckˆ = 0 | A || B | cos θ = 0
cos θ = 0
10 + 14 + 3c = 0
θ = 90°
24 = – 3c
c = –8 4. When two vectors A and B of magnitude a
2. The resultant of the vectors A and B depends and b are added, the magnitude of the resultant
also on the angle θ between them. The vector is always
magnitude of the resultant is always given by (a) equal to (a + b)
(a) A + B + 2AB cos θ (b) less than (a + b)
(c) greater than (a + b)
(b) ( A + B + 2AB cos θ ) (d) not greater than (a + b)
EAMCET-1993
(c) A + B + 2AB cos θ
2 2
Ans. (d) : Given,
( )
2
(d) A 2 + B2 + 2ABcos θ | A |= a, B = b
EAMCET-1992 | A + B |= a 2 + b 2 + 2ab cos θ
Ans. (c) :
| A + B |max = a 2 + b 2 + 2ab [For max, θ = 0]
| A + B |max = (a + b)
Hence, magnitude of resultant vector is not greater than
(a + b)
5. If a unit vector is represented by
0.5iˆ + 0.8jˆ + ckˆ , the value of c is
(a) 1 (b) 0.11
Resultant vector, R (c) 0.011 (d) 0.39
From ∆DOF, TS-EAMCET-10.09.2020, Shift-1
(OD)2 =(OF)2 + (DF)2 EAMCET-1994
(OD)2 = (OE + EF)2 + (DF)2 Ans. (b) : Â = 0.5iˆ + 0.8ˆj + ckˆ
R 2 = ( A + B cosθ)2 + ( B sinθ)2 ˆ |= 1
∴ |A
= A2 + B2 cos2θ + 2AB cosθ + B2 sin2θ
= A2 + B2 (cos2θ + sin2θ) + 2AB cosθ 0.52 + 0.82 + c2 = 1
R = A2 + B2 + 2AB cosθ
2
c2 = 0.11
R = A 2 + B2 + 2ABcos θ c = 0.11
Objective Physics Volume-I 345 YCT
6. The angle made by the vector A = ˆi + ˆj with x- Ans. (c) : Given,
axis is A = a x ˆi + a y ˆj + a z kˆ
(a) 90º (b) 45º Let B = ˆi − ˆj
(c) 22.5º (d) 30º Component of vector A along any vector B
EAMCET-1996
A.B
Ans. (b) : Given that, =
A = ˆi + ˆj B
Component of vector A = a x ˆi + a y ˆj + a z kˆ along
| A |= 12 + 12 = 2
Ax = 1, Ay = 1 B = ˆi − ˆj ( )
If θ is the angle made by the vector with x–axis than,
A 1 =
( a ˆi + a ˆj + a kˆ ) .( ˆi − ˆj) a
x y z
= x − ay
cos θ = x ⇒ cos θ =
(1) + ( −1)
2 2
|A| 2 2
θ = 45° 9. For vectors A and B making an angle θ which
7. The angle between two vectors 6iˆ + 6jˆ - 3kˆ and one of the following relations is correct?
(a) A × B = B × A (b) A × B = ABsin θ
7iˆ + 4jˆ + 4kˆ is given by
(c) A × B = ABcos θ (d) A × B = − B × A
−1  1  −1  5  DCE-2009
(a) cos   (b) cos  
 3   3  Ans. (d) : We know that,
Cross product of vectors A and B
 2  −1
 5 
(c) sin −1   (d) sin   A × B = ABsin θ
 3 
 3   Cross product of vectors B and A
EAMCET-1999 B× A = −BA sin θ
Ans. (d) : Given that,
So, A × B = −B× A
A = 6iˆ + 6ˆj − 3kˆ
10. Given two vectors A = −ˆi + 2jˆ − 3kˆ and
B = 7iˆ + 4ˆj + 4kˆ
B = 4iˆ − 2jˆ + 6kˆ . The angle made by (A + B)
A ⋅ B =| A || B | cos θ with x-axis is
( )( )
6iˆ + 6ˆj − 3kˆ ⋅ 7iˆ + 4ˆj + 4kˆ (a) 30°
(c) 60°
(b) 45°
(d) 90°
=  36 + 36 + 9 ⋅ 49 + 16 + 16  cos θ AP EAMCET(Medical)-2007
 
Ans. (b) : A = −ˆi + 2ˆj − 3kˆ
42 + 24 −12 = 81 81.cos θ = 9×9cos θ
54 = 81cos θ B = 4iˆ − 2jˆ + 6kˆ
54 A + B = (−ˆi + 2jˆ − 3k)ˆ + (4iˆ − 2ˆj + 6k)
ˆ
cos θ =
81 A + B = 3iˆ + 0ˆj + 3kˆ
cosθ = =
6 2 α is angle with x–axis
9 3 x − component of A + B ( )
  cosα =
2
sin θ = 1− cos θ = 1−  
2 2
2
  |A+B|
 3
3 3
2 4 5 cosα = =
sin θ = 1− = 9+0+9 3 2
9 9 1
5 cosα =
sinθ = 2
3 α = 45o
5 11. Of the vectors given below, the parallel vectors
θ = sin −1 are,
3

8. The component of vector A = a ˆi + a ˆj + a kˆ
x y z
A = 6iɵ + 8jɵ

along the direction of ˆi - ˆj is B = 210iɵ + 280kɵ
(a) ax – ay + az (b) ax – ay →
C = 5.1iɵ + 6.8jɵ
( )
(c) a x − a y / 2 (d) (ax + ay + az)

EAMCET-2008 D = 3.6iɵ + 6jɵ + 48kɵ

Objective Physics Volume-I 346 YCT


→ → → →
(a) A and B (b) A and C Ans. (a): A = 3 ˆi + ˆj
→ → → → Angle with x–axis :
(c) A and D (d) C and D
A
AP EAMCET(Medical)-2006 cosθ = x
Ans. (b) : If component of vector is same then vectors | A|
will be same.
3 3
A = 6iˆ + 8jˆ cosθ = =
( ) 2
2
3 + (1)
2
B = 210iˆ + 280kˆ
ˆ ˆ
C = 5.1i + 6.8j θ = 30°
D = 3.6iˆ + 6ˆj + 48kˆ π
θ=
6
∵ C = 5.1iˆ + 6.8jˆ =
2
(
1.7 ˆ ˆ
)
6i + 8j 14. The unit vector parallel to resultant of the

Hence, it is clear that A and C are parallel and we can vectors A = 4iˆ + 3jˆ + 6kˆ and B = −ˆi + 3jˆ − 8kˆ is:
1.7
write as, C =
2
A (a)
1 ˆ ˆ
7
(
3i + 3j − 2kˆ ) (b)
7
(
1 ˆ ˆ
)
3i + 6j − 2kˆ
This implies that A is parallel to C.
12. A vector Q which has a magnitude of 8 is (c)
1 ˆ ˆ
49
(
3i + 6 j − 2kˆ ) (d)
1 ˆ ˆ
49
(
3i − 6 j + 2kˆ)
added to the vector P , which lies along the X- AP EAMCET(Medical)-2000
axis. The resultant of these two vectors is a Ans. (b) : A = 4iˆ + 3jˆ + 6kˆ
third vector R , which lies along the Y-axis and
B = −ˆi + 3jˆ − 8kˆ
has a magnitude twice that of P . The
magnitude of P is: R = A + B = (4iˆ + 3jˆ + 6k) ˆ + (−ˆi + 3jˆ − 8k)ˆ
6 8 R = 3iˆ + 6ˆj − 2kˆ
(a) (b)
5 5
R
12 16 unit vector, R̂ =
(c) (d) |R|
5 5
AP EAMCET(Medical)-2004 | R |= 3 + 6 + 22
2 2

Ans. (b) : Given,


= 9 + 36 + 4
Q = 8 units
= 49
R = 2P
| R |= 7
3iˆ + 6jˆ − 2kˆ
R̂ =
7
1
(
R̂ = 3iˆ + 6ˆj − 2kˆ
7
)
15. Pressure is a scalar quantity because
(a) it is the ratio of force to area and both force
and area are vector quantities
Q2 = R 2 + P2 (b) it is the ratio of magnitude of force to area
(8) 2 = (2P) 2 + P 2 (c) it is the ratio of component of the force
normal to the area
= 4P 2 + P 2 (d) it depends on the size of the area chosen
64 = 5P 2 SCRA-2015
64 8 Ans. (c) : Pressure is a scalar quantity because pressure
P =
2
⇒P=
5 5 is the ratio of normal force to the area and the direction
13. Angle (in rad) made by the vector 3 ˆi + ˆj with of force is not required.
the X-axis: 16. The component of a vector r along X-axis will
π π have a maximum value, if :
(a) (b) (a) r is along positive X-axis
6 4 (b) r is along positive Y-axis
π π
(c) (d) (c) r is along negative Y-axis
3 2 (d) r makes an angle of 450 with the X-axis
AP EAMCET(Medical)-2005
Karnataka CET-2016
Objective Physics Volume-I 347 YCT
Ans. (a) : rx = r cosθ Ans. (c) : A.B = 0 ⇒ A ⊥ B
rx is maximum when θ = 0
r will be along positive X-axis for maximum value. A.C = 0 ⇒ A ⊥ C
17. Which of the following is not a vector quantity? A is perpendicular to both B and C and B × C is also
(a) Weight (b) Nuclear spin perpendicular to both B and C .
(c) Momentum (d) Potential energy
Karnataka CET-2014 Therefore, A is parallel to B × C
Ans. (d) : Weight, Nuclear spin and Momentum are 21. Three forces F1, F2 and F3 together keep a body
vector quantities because they have both magnitude as in equilibrium. If F1 = 3 N along the positive x-
well as direction, whereas potential energy has axis, F2 = 4 N along the positive y-axis, then the
magnitude only but no direction, thus it is a scalar third force F3 is
quantity. 3
18. Two equal forces (P each) act at a point (a) 5N making an angle θ = tan −1   with
4
inclined to each other at an angle of 1200. the
magnitude of their resultant is : negative y-axis
(a) P/2 (b) P/4 4
(b) 5N making an angle θ = tan −1   with
(c) P (d) 2P 3
Karnataka CET-2004 negative y-axis
Ans. (c) : Given, Q = P, θ = 120° 3
(c) 7N making an angle θ = tan −1   with
∵ R = P 2 + Q 2 + 2PQ cos θ 4
= P + P + 2P × P cos120°
2 2 negative y-axis
4
 1 (d) 7N making an angle θ = tan −1   with
= P + P −  2P × P ×  = P
2 2
3
 2
negative y-axis
19. The resultant of two forces 3P and 2P is R. If J&K CET- 2010
the first force is doubled then the resultant is
also doubled. The angle between the two forces Ans. (c) : F1, F2, F3 keep a body in equilibrium then
is : resultant of force,
(a) 900 (b) 1800
(c) 600 (d) 1200
Karnataka CET-2001
Ans. (d) : Case – I
R= ( 3P )2 + ( 2P )2 + 12P 2cosθ
R = P 13 + 12cos θ
Case -II
R1 = ( 6P )2 + ( 2P )2 + 24P 2 cos θ
= P 40 + 24cos θ ΣF = 0
From case (I) and case (II), F1 + F2 +F3 = 0
R1 40 + 24 cosθ 3 + 4 + F3 = 0
= =2 [∵ R1 = 2R ] F3 = – 7N
R 13 + 12cosθ
40 + 24 cosθ = 52 + 48 cosθ Magnitude of F3 = 7N
24 cosθ = –12 θ is angle made with-Y axis
1  1 3
cosθ = − ⇒ θ = cos −1  −  tanθ =
2  2 4
θ= 1200 θ = tan–1 3/4
F3 make angle with negative y-axis.
20. Three vectors satisfy the relation A.B = 0 and
22. Magnitudes of four pairs of displacement
A.C = 0 then A is parallel to : vectors are given. Which pair of displacement
(a) C (b) B vectors, under vector addition, fails to give a
(c) B × C (d) B.C resultant vector of magnitude 3 cm ?
JCECE-2013 (a) 2 cm, 7 cm (b) 1 cm, 4 cm
COMEDK 2013 (c) 2 cm, 3 cm (d) 2 cm, 4 cm
Karnataka CET-2003 J&K CET- 2009
Objective Physics Volume-I 348 YCT
Ans. (a) : The magnitude R of the resultant of two 26. Two vectors are given by A = 3iˆ + ˆj + 3kˆ and
vectors A and B depends upon the magnitudes of A and
B and the angle θ between them and is given by B = 3iˆ + 5jˆ - 2kˆ . Find the third vector C if
R2 = A2 + B2 + 2AB cos θ A + 3B - C = 0
When θ = 0, R is maximum and given by (a) 12iˆ +14jˆ +12kˆ (b) 13iˆ +17jˆ +12kˆ
R 2max = A2 + B2 + 2AB = (A + B)2
(c) 12iˆ +16jˆ - 3kˆ (d) 15iˆ +13jˆ + 4kˆ
Rmax = A + B
J&K CET- 2007
When θ = 180º, R is minimum and given by
R 2min = A2 + B2 − 2AB = (A − B)2 Ans. (c) : Given, A = 3i + j + 3k , B = 3i + 5ˆj − 2kˆ
ˆ ˆ ˆ ˆ

Rmin = A – B A + 3B − C = 0
Thus, the magnitude of resultant will lie between A – B
and A + B.
( )
3iˆ + ˆj + 3kˆ + 3 3iˆ + 5ˆj − 2kˆ − C = 0
Now, 3iˆ + ˆj + 3kˆ + 9iˆ + 15jˆ − 6kˆ − C = 0
Checking option (a)
12iˆ + 16ˆj − 3kˆ − C = 0
|A – B| = |2 – 7|= 5
|A + B| = |2 + 7| = 9 C = 12iˆ + 16ˆj − 3kˆ
So, 5 ≤ R ≤ 9 and R = 4 27. Vector which is perpendicular to a
Hence, the option (a) is the correct answer.
(acos θ ˆi + bsin θ ˆj) is
23. A body is under the action of two mutually
1 1
perpendicular forces of 3N and 4N. The (a) bsinθiˆ − a cosθ ˆj (b)sinθiˆ − cosθ ˆj
resultant force acting on the body is a b
(a) 7 N (b) 1 N (c) 5kˆ (d) all of these
(c) 5 N (d) zero J&K CET- 2006
J&K CET- 2008 Ans. (d) : Two vectors are perpendicular if their dot
Ans. (c) : The two forces be product is zero i.e., A ⋅ B = 0.
A = 3N and B = 4N
In option (a)
A is mutually perpendicular to B.
∴ θ = 90° ( )(
a cos θˆi + bsin θˆj ⋅ bsin θˆi − a cos θˆj )
R = A 2 + B2 + 2ABcos θ = ab cos θ sin θ − absin θ cos θ = 0
In option (b)
R = 42 + (3) 2 + 2ABcos 90° 1 1
(a cos θˆi + bsin θˆj) ⋅ ( sin θˆi − cos θˆj)
R = 16 + 9 + 0 a b
R = 5N = sin θ cos θ − sin θ cos θ=0
24. If the scalar and vector products of two vectors In option (c)
A,B are equal in magnitude, then the angle (a cos θˆi + bsin θˆj) ⋅ 5kˆ = 0. ( )
between the two vectors is
28. Velocity is
(a) 45° (b) 90°
(a) scalar
(c) 180° (d) 360° (b) vector
J&K CET- 2008 (c) neither scalar nor vector
Ans. (a) : A.B = A × B (d) both scalar and vector
|A||B|cosθ = |A||B|sinθ J&K CET- 2002
sin θ A B Ans. (b) : A vector quantity is defined as the physical
= quantity that has both magnitude as well as direction.
cos θ A B
tanθ = 1 Velocity is the directional speed of a object in motion
and indication of rate of change in position as observed
θ = 45° by a particular frame of reference.
25. A is a vector with magnitude A, then the unit Velocity is a physical vector quantity.
vector  in the direction of A is 29. The sum of two vectors A and B is at right
(a) AA (b) A ⋅ A angles to their difference. Then
(a) A = B
A
(c) A × A (d) (b) A = 2B
A (c) B = 2A
J&K CET- 2008 (d) A and B have the same direction
A A BCECE-2008
Ans. (d) : Unit vector  = = J & K CET - 1998
|A| A UP CPMT - 2006
Objective Physics Volume-I 349 YCT
Ans. (a) : Let r1 and r2 be the sum and difference of 32. The sum of two vectors A and B is at right
vectors A and B respectively i.e., angles to their difference. This is possible if
r1 = A + B (a) A = 2B (b) A = B
r2 = A – B (c) A =3B (d) B =2A
r1 is perpendicular to r2 (given) J&K CET- 1998
Taking the dot product of r1 and r2 Ans. (b) : Let, P1 and P2 sum and difference of vectors
r 1. r 2 = ( A + B ) . ( A – B )
0 = A2 – B2 A and B ,
A2 = B2 P1 = ( A + B )
A=B
P2 = ( A – B )
30. The vectors A and B are such that
A+B = A–B P1 . P2 = ( A + B ).( A – B )
The angle between the two vectors is 0 = A2 – B2
(a) 60º (b) 75º A2 = B2
(c) 45º (d) 90º A=B
WBJEE-2016, 33. What is the torque of a force 3iˆ + 7jˆ + 4kˆ about
AIIMS-25.05.2019(E) Shift-2
J&K CET- 2003, 1999 the origin, if the force acts on a particle whose
position vector is 2iˆ + 2jˆ + 1kˆ ?
Ans. (d) : Let angle between A and B be θ
The resultant of A + B is given by (a) ˆi – 5jˆ + 8kˆ (b) 2iˆ + 2jˆ + 2kˆ

R = A 2 + B2 + 2ABcos θ (c) ˆi + ˆj + kˆ (d) 3iˆ + 2jˆ + 3kˆ


The resultant of A − B is given by J&K-CET-2014
R ' = A + B − 2ABcos θ
2 2 Ans. (a) : Given that,
According to the question, F = 3iˆ + 7ˆj + 4kˆ
R = R'
r = 2iˆ + 2ˆj + 1kˆ
A 2 + B2 + 2ABcos θ = A 2 + B2 − 2ABcos θ
τ = r×F
A 2 + B2 + 2ABcos θ = A 2 + B2 − 2ABcos θ
4ABcos θ = 0 ˆi ˆj kˆ
θ = 90° τ= 2 2 1
31. The vector sum of two forces is perpendicular 3 7 4
to their vector differences. In that case, the
forces τ = î (8– 7) – ĵ (8 – 3) + k̂ (14 – 6)
(a) are not equal to each other in magnitude
(b) cannot be predicted τ = î – 5jˆ + 8kˆ
(c) are equal to each other 34. The scalar product of two vectors
(d) are equal to each other in magnitude A = 2iˆ + 2jˆ – kˆ and B = – ˆj + kˆ is given by
[AIPMT 2003]
AIIMS- 2012 (a) A . B = 3 (b) A . B = 4
TS-EAMCET - 09.09.2020 (c) A . B = – 4 (d) A . B = – 3
Ans. (d) : Let f1 and f 2 be the two forces J&K-CET-2013
Then sum of forces, a = f1 + f 2 Ans. (d) : Given that,
A = 2iˆ + 2ˆj − k,
ˆ B = −ˆj + kˆ = 0iˆ − ˆj + kˆ
And difference, b = f1 – f 2

( )
The two forces are perpendicular to each other a.b = 0 ( )(
A.B = 2iˆ + 2ˆj − kˆ . 0iˆ − ˆj + kˆ )
= 0 + 2 × (–1) + (−1) × 1
( f + f ).( f
1 2 1 )
− f2 = 0 = –2 – 1 = –3
| f1 |2 − | f 2 |2 = 0 35. The velocity vector of the motion described by
the position vector of a particle r = 2tiˆ + t 2 ˆj is
| f1 |2 =| f 2 |2
(a) v = 2iˆ + 2t ˆj (b) v = 2tiˆ + 2t ˆj
| f1 |=| f 2 |
In that case both the force are equal and have same (c) v = tiˆ + t 2 ˆj (d) v = 2iˆ + t 2 ˆj
magnitude. J&K-CET-2013
Objective Physics Volume-I 350 YCT
Ans. (a) : Given, r = 2tiˆ + t 2 ˆj 38. Two forces each of magnitude 'P' act at right
angles. Their effect is neutralized by a third
Velocity ( v ) = drdt force acting along their bisector in opposite
direction. The magnitude of the third force is
dr  π 
= 2iˆ + 2tjˆ cos 2 = 0
dt  
36. A certain vector in the xy plane has an x- P
component of 12 m and a y-component of 8 m. (a) P (b)
2
It is then rotated in the xy plane so that its x-
component is halved. Then its new y- P
(c) 2P (d)
component is approximately 2
(a) 14 m (b) 13.11 m MHT-CET 2020
(c) 10 m (d) 2.0 m Ans. (c) : The third force will have magnitude equal to
their resultant,
J&K-CET-2012
Ans. (b) : x – component = 12cm
y – component = 8cm
Length of the resultant vector (R)
= x 2 + y 2 = 122 + 82
= 144 + 64 = 208
Now,
x 12
x' = = = 6cm
2 2
Resultant will always be constant even after the rotation R= R 12 + R 22 + 2R 1R 2 cos θ
So,
R= P 2 + P 2 + 2.P.P cos90°
(x ') + (y ') = 208
2 2

R = 2P 2
(6)2 + (y ')2 = 208
R = 2P
(y')2 = 208 – 36 = 172
39. In the given diagram, if PQ = A, QR = B and
(y') = 172 RS = C, then PS equals
(y') = 13.11m
37. Figure shows three forces F1 , F2 and F3 acting
along the sides of an equilateral triangle. If the
total torque acting at point 'O' (centre of the
triangle) is zero then the magnitude of F3 is
(a) A – B + C (b) A + B – C
(c) A + B + C (d) A – B – C
(e) – A – B – C
Kerala CEE - 2016
Ans. (c) : Given, PQ = A, QR = B, RS = C, PS = ?

(a) F1+F2 (b) F1-F2


F −F F
(c) 1 2 (d) 1
2 F2
According to polygon law of vector addition,
MHT-CET 2020 PQ + QR + RS – PS = 0
Ans. (a) : Force F1 and F2 produce anticlockwise PS = PQ + QR + RS
PS = A + B + C
torque while force F3 produces clockwise torque. The
40. Among the following, the vector quantity is
torque in the two directions balance each other. The (a) pressure (b) gravitation potential
perpendicular distance of the forces from the centre is (c) stress (d) impulse
the same. (e) distance
∴ F1r + F2r = F3r or F1+F2 = F3 Kerala CEE 2012
Objective Physics Volume-I 351 YCT
Ans. (d) : Impulse is defined as the product of net force 43. A particle acted upon by constant forces
and time interval for which it was applied. 4iɵ + ɵj - 3kɵ and 3iɵ + ɵj - kɵ is displaced from the
i.e. Impulse = Ft
Force is vector quantity. Therefore, Impulse is a vector point ɵi + 2jɵ + 3kɵ to the point 5iɵ + 4jɵ + k. ɵ The
quantity. Stress is a tensor quantity. total work done by the forces in SI unit is
41. The sum of magnitudes of two forces acting at (a) 20 (b) 40
(c) 50 (d) 30
a point is 16 N and their resultant 8 3 N is at (e) 35
90º with the force of smaller magnitude. The Kerala CEE - 2008
two forces (in N) are Ans. (b) : Given,
(a) 11, 5 (b) 9, 7
(c) 6, 10 (d) 4, 12 F1 = 4iˆ + ˆj − 3kˆ
(e) 2, 14 F2 = 3iˆ + ˆj − kˆ
Kerala CEE 2012
Ans. (e) : Given, r1 = ˆi + 2ˆj + 3kˆ
P + Q = 16 r2 = 5iˆ + 4ˆj + kˆ
R=8 3 Now force, FR = F1 + F2
∵ R = P 2 + Q 2 + 2PQ cos θ
(
F = 4iˆ + ˆj – 3kˆ + 3iˆ + ˆj – kˆ
R ) ( )
8 3 = P 2 + Q 2 + 2PQcos θ
FR = 7iˆ + 2ˆj − 4kˆ
sin θ
∵ tan 90o = =∞ Displacement, dr = r2 − r1
P + Qcos θ
P dr = (5iˆ + 4ˆj + k)
ˆ − (iˆ + 2ˆj + 3k)
ˆ
P + Q cos θ = 0 ⇒ cos θ = −
Q dr = 4iˆ + 2ˆj − 2kˆ
So, Work done by the forces, W = FR .dr
 P = (7iˆ + 2ˆj − 4k).(4i
ˆ ˆ + 2ˆj − 2k)
ˆ
8 3 = P 2 + Q 2 + 2PQ  − 
 Q (where, ˆi ⋅ ˆi = ˆj ⋅ ˆj = kˆ ⋅ kˆ = 1 )
8 3 = P + Q − 2P ⇒ 8 3 = Q − P
2 2 2 2 2
W = 28 + 4 + 8
Q − P = 192
2 2 W = 40J
∴ Q2 – (16 – Q)2 = 192 ( ∵ P = 16 – Q) the two vectors A = 2iɵ + 3jɵ + 4kɵ and
44. If
Q2 − 256 − Q 2 + 32Q = 192 B = ɵi + 2jɵ - nkɵ are perpendicular then the value
32Q = 448 of n is :
Q = 14 N (a) 1 (b) 2
Now, from P + Q = 16 (c) 3 (d) 4
P = 16 – 14 = 2N (e) 5
P = 2N Kerala CEE 2006
Q = 14N Ans. (b) : Given,
42. A force (4iɵ + ɵj - 2k)ɵ N acting on a body A = 2iˆ + 3jˆ + 4kˆ
maintains its velocity at ( 2iˆ + 2jˆ + 3kˆ ) ms–1. The B = ˆi + 2ˆj − nkˆ
If two vectors are perpendicular then their scalar
power exerted is product is zero.
(a) 4 W (b) 5 W
(c) 2 W (d) 8 W ∴ A.B = 0
(e) 1 W ( 2iˆ + 3jˆ + 4kˆ ) ⋅ ( ˆi + 2ˆj − nkˆ ) = 0
Kerala CEE - 2010 2 + 6 − 4n = 0
Ans. (a) : Given, n=2
(
F = 4iˆ + ˆj – 2kˆ ) 45. A particle is displaced from a position
v = (2iˆ + 2jˆ + 3kˆ )
ɵ to
(2iɵ - ɵj + k) another ɵ
(3iɵ + 2jɵ - 2k)
position
ɵ The
under the action of the force of (2iɵ + ɵj - k).
We know that,
P = F⋅v work done by the force in an arbitrary unit is:
(a) 8 (b) 10
( )(
P = 4iˆ + ˆj – 2kˆ ⋅ 2iˆ + 2jˆ + 3kˆ ) (c) 12 (d) 16
(where, ˆi ⋅ ˆi = ˆj ⋅ ˆj = kˆ ⋅ kˆ = 1 ) (e) 20
JCECE-2018
P = 8 + 2 − 6 ⇒ P = 4W Kerala CEE 2005
Objective Physics Volume-I 352 YCT
Ans. (a) : Given that, 48. The power utilised when a force of
F = 2iˆ + ˆj − kˆ (2iˆ + 3jˆ + 4k)N
ˆ acts on a body for 4s,
r1 = 2iˆ − ˆj + kˆ producing a displacement of (3iˆ + 4jˆ + 5k)
ˆ m, is
r2 = 3iˆ + 2ˆj − 2kˆ (a) 9.5 W (b) 7.5 W
(c) 6.5 W (d) 4.5 W
W = F.dr AP EAMCET (21.09.2020) Shift-I
W = (2iˆ + ˆj − k).(r
ˆ
2 − r1 ) ( )
∵ dr = r 2 − r1 Ans. (a) : Power = F.v
( ˆ ˆ ˆ)(  ˆ ) (
ˆ ˆ ˆ )
= 2i + j – k ⋅  3i + 2j – 2k − 2i − j + k 

ˆ ˆ 
 And, v=
d
t
= (2iˆ + ˆj − k).(i
ˆ ˆ + 3jˆ − 3k) ˆ =2+3+3
3iˆ + 4ˆj + 5kˆ
∴ W = 8 unit v=
4
46. If a vector A is given as A = 4iɵ + 3jɵ + 12kɵ , then ˆ ˆ ˆ
the angle subtended with the x-axis is ∴ Power = (2iˆ + 3jˆ + 4k).ˆ (3i + 4 j + 5k)
4
4 3
(a) sin −1   (b) sin −1   3
= 2 × + 3× + 4 ×
4 5
13  13  4 4 4
3 4 6 12 20 6 + 12 + 20
(c) cos −1   (d) cos −1   = + + =
13  13  4 4 4 4
COMEDK-2019 38
UPSEE - 2015 =
4
Ans. (d) : Given, A = 4iˆ + 3jˆ + 12kˆ Power = 9.5 W
Let θ be the angle made by vector A with X-axis. 49. A = 4iˆ + 3jˆ and B = 4iˆ + 2j. ˆ Find a vector
Ax
∴ cos θ = parallel to A but has magnitude five times that
A
of B.
( ) ( )
4 4
cos θ = = (a) 20 2iˆ + 3jˆ (b) 20 4iˆ + 3jˆ
4 + 3 + 12
2 2 2 13
4
θ = cos −1  
(
(c) 20 2iˆ + ˆj ) ( )
(d) 10 2iˆ + ˆj
 13  BITSAT-2007
47. The angle between two vectors A and B is θ. Ans. (b) : Given, A = 4i + 3j , B = 4i + 2 j ˆ ˆ ˆ ˆ
Vector R is the resultant of the two vectors. If
A 4iˆ + 3jˆ
θ Â = =
R makes an with A, then A 42 + 32
2
B
(a) A = 2B (b) A =
2 (
1
 = 4iˆ + 3jˆ
5
)
(c) A = B (d) AB = 1 ˆ
UPSEE - 2013 P = 5 B A
Ans. (c) : The angle α which the resultant R makes 1
= 5 × 42 + 22 × (4iˆ + 3j)
ˆ
with A is given by– 5
Bsin θ θ P = 20(4iˆ + 3j)
ˆ
tan α = Here, α =
A + Bcos θ 2 50. Given that A + B = R and A2 + B2 = R2 The
θ Bsin θ
Hence, tan = angle between Aand B is
2 A + Bcos θ
(a) 0 (b) π/4
θ
sin (c) π/2 (d) π
2 = 2Bsin(θ / 2) cos(θ / 2) BITSAT-2009
θ A + Bcos θ
cos Ans. (c) : Given, A + B = R and A + B = R2
2 2
2 R2 = A2 + B2 + 2AB cosθ
A + Bcos θ = 2Bcos 2 (θ / 2) Where, θ = Angle between A & B,
A + B[2cos 2 (θ / 2) − 1] = 2Bcos 2 (θ / 2) ∵ R2 = A2 + B2
θ θ ∴ 2
A + B2 = A2 + B2 + 2AB cosθ
A + 2Bcos 2   − B = 2Bcos 2  
2 2 ∴cosθ = 0
A–B=0 π
∴A=B θ=
2
Objective Physics Volume-I 353 YCT
51. The two vectors A and B are drawn from a Ans. (b) : R = xiˆ + yjˆ + zkˆ
common point and C = A + B, then angle A = 3iˆ + 2ˆj + 5kˆ
between A and B is A ⋅ R = (xiˆ + yjˆ + zk)
ˆ ⋅ (3iˆ + 2ˆj + 5k)
ˆ
(1) 90° if C2 = A2 + B2
(2) greater than 90° if C2 < A2 + B2 A ⋅ R = 3x + 2y + 5z
(3) greater than 90° if C2 > A2 + B2 ∇(A ⋅ R) =
d(3x) ˆ d(2y) ˆ d(5z) ˆ
i+ j+ k
(4) less than 90° if C2 > A2 + B2 dx dy dz
Correct options are- ∇(A ⋅ R) = 3iˆ + 2ˆj + 5kˆ
(a) 1,2 (b) 1, 2, 3, 4
(c) 2, 3, 4 (d) 1, 2, 4 ∇(A ⋅ R) = A
BITSAT-2011 54. The direction of A is vertically upward and
Ans. (d) : C = A + B direction of B is in north direction. The
∵ C2 = A2 + B2 + 2AB cosθ direction of A × B will be
(a) Western direction
C 2 − ( A 2 + B2 ) (b) Eastern direction
cosθ =
2AB (c) At 45º upward in north
Case-I, C2 = A2 + B2 (d) Vertically downward
A 2 + B2 − ( A 2 + B2 ) CG PET- 2009
cosθ = =0 Ans. (a) :
2AB
θ = π/2 or 90°
Case-II, C2 < A2 + B2
C 2 − ( A 2 + B2 )
cosθ = (–ve)
2AB
Then, θ > 90°
Case-III, C2 > A2 + B2
C 2 − ( A 2 + B2 ) Considering vertically upward direction as z-axis and
cosθ = (+ve) north direction as y-axis.
2AB
Then, θ < 90° A = akˆ , B = bjˆ

52. Given P = 2iˆ – 3jˆ + 4kˆ and Q = ˆj – 2k. ˆ The ∴ A × B = akˆ × bjˆ = ab(−ˆi)
Thus, it is along negative x-axis
magnitude of their resultant is
∴ A × B is along west.
(a) 3 (b) 2 3
(c) 3 3 (d) 4 3 55. If A = B + C and the values of A, B and C are
BITSAT -2018 13, 12 and 5 respectively, then the angle
between A and C will be
Ans. (b) : Given, P = 2i − 3j + 4k , Q = j − 2kˆ
ˆ ˆ ˆ ˆ
(a) cos −1 ( 5 /13) (b) cos −1 (13 /12 )
R = P+Q
(c) π / 2 (d) sin −1 ( 5 /12 )
= (2iˆ − 3jˆ + 4k)
ˆ + (ˆj − 2k)
ˆ
CG PET- 2009
R = 2iˆ − 2ˆj + 2kˆ Ans. (a) : Given that, A = 13,B = 12,C = 5
R = (2) + (−2) + (2)
2 2 2 A2 = B2 + C2 + 2BC cosθ
(13)2 = (12)2 + (5)2 + 2 × 12 × 5 cosθ
R =2 3 cos θ = 0
53. The position vector of a point is θ = 90°
Hence, it is a right-angle triangle.
R = xiˆ + yjˆ + zkˆ and another vector is
A = 3iˆ + 2jˆ + 5kˆ . Which of the mathematical
relation is correct?
(a) ∇ A ( )
ˆ Rˆ = 0

( )
(b) ∇ A R = A

(c) ∇ (A R ) = R So, angle between A and C is,


5 5
(d) None of these cos α = ⇒ α = cos −1  
CG PET- 2009 13  13 

Objective Physics Volume-I 354 YCT


56. If r1 = 2 xˆ , r2 = 2y,ˆ where x̂ and ŷ are unit Ans. (b) : r = α cos ωtiˆ + α sin ωtjˆ
vectors along the X-axis and Y-axis
dr
respectively, then the magnitude of r1 + r2 is v= = (−α sin ωt ) ωˆi + (α cos ωt ) ωˆj
(a) 2 2 (b) 2 3 dt
v.r = (−αω sin ωt)iˆ + (αω cos ωt)ˆj .  (α cos ωt)iˆ + (α sin ωt)ˆj
(c) 3 2 (d) 3    
TS-EAMCET.14.09.2020, Shift-2 = −α 2 ω sin ωt.cos ωt + α 2 ω cos ωt.sin ωt
Ans. (a) : Given, r1 = 2x, ˆ r2 = 2yˆ v.r = 0
r1 + r2 = 2x + 2y
ˆ ˆ ∵ Dot product of v and r is zero
Magnitude, r1 + r2 = 22 + 22 = 2 2 ∴ Both are perpendicular to each other.
57. Let A1 + A2 = 5A3, A1 – A2 = 3A3, 60. If a vector A having a magnitude of 8 is added
| A | to a vector B which lies along x-axis, then the
A 3 = 2iˆ + 4j,
ˆ then 1 is
resultant of two vectors lies along y-axis and
| A2 |
(a) 4 (b) 8 has magnitude twice that of B. The magnitude
(c) 2 (d) 6 of B is
TS-EAMCET.14.09.2020, Shift-2 6 12
(a) (b)
Ans. (a) : Given, A1 + A2 = 5A3 ...(1) 5 5
A1 – A2 = 3A3 ...(2) 16 8
Adding equation (1) & (2), (c) (d)
2A1 = 8A3 ⇒ A1 = 4A3 5 5
JCECE-2012
∴ A1 = 8iˆ + 16ˆj
Ans. (d) : Given,
Subtracting equation (2) from (1),
2A2 = 2A3
∴ A 2 = A 3 = 2iˆ + 4ˆj
A1 82 + 162
= = 16 = 4
A2 22 + 42
58. The magnitude of x and y components of A are
7 and 6 respectively. Also the magnitudes of x
and y components of A+B are 11 and 9
respectively. Calculate the magnitude of vector A = 8 units
B. Since, B is along x - axis and resultant of two vector
(a) 10 (b) 5 C lies on y - axis
(c) 6 (d) 3
JCECE-2018 So, B and C are perpendicular vector.
Ans. (b) : Given, A = 7iˆ + 6ˆj Hence,
| A |2 =| B |2+ | C |2
| 8 |2 =| B |2+2B |2
Let, B = b1ˆi + b 2 ˆj
64 = 5B2
64
B2 =
5
8
∴ 7 + b1 = 11 ⇒ b1 = 4 ∴ B=
5
6 + b2 = 9 ⇒ b2 = 3
61. It two forces each of 2N are inclined at 60º,
∴ B = 4iˆ + 3jˆ
then resultant force is:
B = 42 + 32 = 25 = 5 (a) 2 N (b) 2 5N
59. The position vector of a particle is (c) 2 3N (d) 4 2N
ˆ ˆ
r = (αcosωt)i + ( αsinωt)j . The velocity vector JCECE-2006
of the particle is Ans. (c) : Let A & B be two forces,
(a) Parallel to position vector ∴ R = A 2 + B2 + 2ABcos θ
(b) Perpendicular to position vector
(c) Directed towards the origin R = 22 + 22 + 2 × 2 × 2cos 60o (cos 60o = 1/2)
(d) Directed away from the origin ∴ R = 12 = 2 3N
JCECE-2014
BCECE - 2004 ∴ R = 2 3N
Objective Physics Volume-I 355 YCT
62. Two vectors have magnitudes 3 and 5. If angle What is the magnitude of the resultant between
between them is 60º, then the dot product of these two vectors?
two vectors will be : (a) 20 3 (b) 35
(a) 7.5 (b) 6.5
(c) 8.4 (d) 7.9 (c) 15 3 (d) 10 3
JCECE-2003 COMEDK 2015
Ans. (d) : The vector representation is as follows,
Ans. (a) : Let A & B be two vector,
∴ A.B = A B cos θ
= 3 × 5 cos60o
1
= 15 ×
2
A ⋅ B = 7.5
63. Calculate the work done when a force ∴ θ = 180 − 30 − 30 = 120
F = 2iˆ + 3jˆ – 5kˆ units acts on a body producing R = A 2 + B2 + 2ABcos θ
a displacement s = 2iˆ + 4jˆ + 3kˆ units : R = 102 + 202 + 2 × 10 × 20 × cos120o
(a) 1 unit (b) 20 unit  1
(c) 5 unit (d) zero R = 100 + 400 − 200 ∵ cos120° = − 
JCECE-2003  2
R = 300
Ans. (a) : Given that, F = 2iˆ + 3jˆ − 5kˆ
s = 2iˆ + 4ˆj + 3kˆ ∴ R = 10 3 Unit
W = F⋅ s 66. Two vectors are given by A = (iˆ + 2jˆ + 2k)
ˆ and
ˆ ˆ ˆ ˆ ˆ ˆ
W = (2i + 3j − 5k).(2i + 4 j + 3k) B = (3iˆ + 6jˆ + 2k)
ˆ . Another vector C has the
W = 4 + 12 − 15 same magnitude as B but has the same
W = 1 Unit
direction as A . Then which of the following
64. Three forces acting on a body are shown in the
figure. To have the resultant force only along vectors represents C ?
the y-direction, the magnitude of the minimum
additional force needed along OX is (a)
3
(
7 ˆ
)
i + 2 ˆj + 2kˆ (b)
7
(
3 ˆ
)
i − 2 ˆj + 2kˆ

(c)
9
(
7 ˆ
)
i − 2 ˆj + 2kˆ (d)
7
(
9 ˆ
)
i − 2 ˆj + 2kˆ
COMEDK 2018
ˆ ˆ ˆ
Ans. (a) : Given that, A = (i + 2 j + 2k)
B = (3iˆ + 6ˆj + 2k)
ˆ
Unit vector along direction of A ,
3
(a) N (b) 13N
4
(c) 0.5 N (d) 1.5 N
COMEDK 2019 And, B = 32 + 6 2 + 2 2
Ans. (c) : ∵ R = (∑ Fx ) + (∑ Fy )
2 2

B = 49 = 7
Let the additional force F be directed along the positive
x-direction. Thus, vector C is–
Taking x-component, the total force should be zero. C= BA ˆ
Let F be the magnitude of minimum force which must
be along x-direction, by resolving the vector we get– 7
1 × cos60° + 2sin30° + F – 4sin30° = 0 C = (iˆ + 2ˆj + 2k)
ˆ
3
1
+1+ F − 2 = 0 67. A particle starts moving from point (2,10,1).
2
F = 1/2 = 0.5 N Displacement for the particle is 8iˆ − 2jˆ + kˆ . The
65. Vector A has a magnitude of 10 units and final coordinates of the particle is
makes an angle of 30° with the positive x-axis. (a) (10, 8, 2) (b) (8, 10, 2)
Vector B has a magnitude of 20 units and (c) (2, 10, 8) (d) (8, 2, 10)
makes an angle of 30° with the negative x-axis. COMEDK 2020

Objective Physics Volume-I 356 YCT

You might also like